Difference between revisions of "2009 AMC 8 Problems/Problem 13"

(See Also)
Line 2: Line 2:
  
 
A three-digit integer contains one of each of the digits <math> 1</math>, <math> 3</math>, and <math> 5</math>. What is the probability that the integer is divisible by <math> 5</math>?
 
A three-digit integer contains one of each of the digits <math> 1</math>, <math> 3</math>, and <math> 5</math>. What is the probability that the integer is divisible by <math> 5</math>?
 
  
 
<math> \textbf{(A)}\  \frac{1}{6}  \qquad
 
<math> \textbf{(A)}\  \frac{1}{6}  \qquad
Line 9: Line 8:
 
\textbf{(D)}\  \frac{2}{3}  \qquad
 
\textbf{(D)}\  \frac{2}{3}  \qquad
 
\textbf{(E)}\  \frac{5}{6}</math>
 
\textbf{(E)}\  \frac{5}{6}</math>
 +
 +
==Solution==
 +
The three digit numbers are <math>135,153,351,315,513,531</math>. The numbers that end in <math>5</math> are divisible are <math>5</math>, and the probability of choosing those numbers is <math>\boxed{\textbf{(B)}\ \frac13}</math>.
  
 
==See Also==
 
==See Also==
 
{{AMC8 box|year=2009|num-b=12|num-a=14}}
 
{{AMC8 box|year=2009|num-b=12|num-a=14}}

Revision as of 16:58, 25 December 2012

Problem

A three-digit integer contains one of each of the digits $1$, $3$, and $5$. What is the probability that the integer is divisible by $5$?

$\textbf{(A)}\  \frac{1}{6}  \qquad \textbf{(B)}\   \frac{1}{3}  \qquad \textbf{(C)}\   \frac{1}{2}  \qquad \textbf{(D)}\  \frac{2}{3}   \qquad \textbf{(E)}\   \frac{5}{6}$

Solution

The three digit numbers are $135,153,351,315,513,531$. The numbers that end in $5$ are divisible are $5$, and the probability of choosing those numbers is $\boxed{\textbf{(B)}\ \frac13}$.

See Also

2009 AMC 8 (ProblemsAnswer KeyResources)
Preceded by
Problem 12
Followed by
Problem 14
1 2 3 4 5 6 7 8 9 10 11 12 13 14 15 16 17 18 19 20 21 22 23 24 25
All AJHSME/AMC 8 Problems and Solutions